LSAT and Law School Admissions Forum

Get expert LSAT preparation and law school admissions advice from PowerScore Test Preparation.

 Administrator
PowerScore Staff
  • PowerScore Staff
  • Posts: 8917
  • Joined: Feb 02, 2011
|
#26253
Complete Question Explanation
(The complete setup for this game can be found here: lsat/viewtopic.php?t=10917)

The correct answer choice is (C)

The stem in this Rule Substitution question requires us to replace the third rule of the game, which created the following relationship:
June15_game_3_#18_diagram_1.png
The rule itself is relatively straightforward. Without it, we are left with the following sequencing relationships:
June15_game_3_#18_diagram_2.png
Our job is to substitute the third rule with a logically equivalent condition, which would have the exact same effect on the order in which the librarians’ duties are scheduled throughout the week. While the wording of the correct answer is difficult-to-impossible to predict, the four incorrect answer choices will either present rules that were not part of the original rule set (also known as Additional Effects conditions), or else they will only partially constrain the variables in the rule being substituted (i.e. Partial Match conditions).

None of the incorrect answer choices contains an Additional Effects condition, as none of them extend beyond the scope of the original rule set. Our task, then, is to determine which four answer choices contain Partial Match conditions.

Answer choice (A) contains a Partial Match. While the rule we are asked to substitute prevents F from being scheduled on Thursday, this inference—by itself—does not have the exact same effect on the order in which the librarians are released. For instance, if all we know is that F cannot be on duty on Thursday, it would be possible to schedule F on any other day (including Friday or Saturday, in direct violation of the original rule set). Because answer choice (A) does not limit F in the same manner as the original rule, it is incorrect.

Answer choice (B) also contains a Partial Match. The inference that either F or H must be on duty on Monday is consistent with our original diagram. However, this rule by itself does not require F to be on duty earlier than K and M. If H were scheduled on Monday, then F could be scheduled on any of the other five days, in direct violation of the rule being replaced. Therefore, answer choice (B) contains a Partial Match, and must be eliminated.

Answer choice (C): This is the correct answer choice. If only H and L can be on desk duty earlier than F, it follows that F must be on duty earlier than all other librarians, including K and M. Therefore, this is not a Partial Match. Answer choice (C) is not an Additional Effects rule either, because the rule we are asked to replace forced F to be earlier than not only K and M, but also G and Z. After all, M :longline: G (second rule) and K :longline: Z (fourth rule). As a result, this answer choice is not overly restrictive, and does not contain an Additional Effects condition. Thus, the effect is identical, and this is the correct answer choice.

Answer choice (D) is attractive, but also contains a Partial Match. The rule we are asked to substitute clearly forces F to be earlier than G and K, as shown in the main diagram. This rule is also attractive because it directly forces F to be earlier than K, which is half of the original rule. Unfortunately, this answer choice does not ensure that F is earlier than M: it only establishes that F is earlier than G. When combined with the second rule, this answer choice presents the following relationship between M, F, and G:
June15_game_3_#18_diagram_3.png
Since the relationship between M and F remains unclear, this answer choice does not restrict the variables in the same manner as the original rule (it allows M to be scheduled on Monday, for instance). Therefore, answer choice (D) contains a Partial Match, and must be eliminated.

Answer choice (E) also contains a Partial Match. The rule we are asked to substitute clearly forces F to be earlier than M and Z, as shown in the main diagram. Thus, there are no Additional Effects produced by answer choice (E). This answer choice may also seem attractive because it replicates the relationship between F and M codified in the original rule. Unfortunately, it fails to ensure that F is earlier than both K and M. The fact that F is earlier than Z does not guarantee that F is also earlier than K, because K—just like F—must be earlier than Z (fourth rule):
June15_game_3_#18_diagram_4.png
Since the original rule requires that F be on duty earlier than K, and this rule does not, answer choice (E) contains only a Partial Match and is incorrect.

Observant test takers will notice that answer choices (D) and (E) contain functionally identical errors: they each satisfy only some of the parameters of the original rule, neglecting to satisfy others.
You do not have the required permissions to view the files attached to this post.

Get the most out of your LSAT Prep Plus subscription.

Analyze and track your performance with our Testing and Analytics Package.